« first day (3788 days earlier)      last day (718 days later) » 

8:59 AM
@XanderHenderson: Repeated poor attempts to solve the same open problem: 1, 2. Can you post a comment or something regarding the policy on open problems? Thanks!
 
9:15 AM
6
Q: Do three consecutive numbers of form $A^2B^3$ exist?

TanakaI (non-mathematician) asked a similar kind of question 5 days ago. Now I revisit the case in a different manner. The powerful numbers may be written in the form $A^2B^3$, where $A$ and $B$ are positive integers. Erdös conjectured in 1975 that there do not exist three consecutive powerful integers...

 
 
3 hours later…
11:54 AM
@user21820 Based on the last meta thread (regarding a Collatz proof), I do not feel empowered to act on those questions. I think that they should be closed, but I don't think that the consensus regarding original research on open problems is as firm as I thought it was.
 
12:15 PM
@MartinR What do you think about this question? I think that the first comment is spot on, and the absence of reaction made me vote to close it.
 
12:34 PM
@JoséCarlosSantos I agree.
 
1:14 PM
This seems like pretty standard fare, but I did not have luck finding a duplicate. Anyone want to take a shot?
https://math.stackexchange.com/q/4459708/29335
 
1:33 PM
I wrote the custom close reason [for that question](https://math.stackexchange.com/questions/4458731/ring-r-and-r-module-m-for-all-4-possibilities-of-noetherian-or-not-should), although I feel the rationale is not on completely solid ground. The quality is not very good to begin with and maybe that's enough for deletion.

I think it's a good example of something that has potential content that's fit to keep, but to achieve that it really should be a better-written self-answered pair. The alternatives of closing and deleting and (even worse) keeping as-is both seem worse.
 
 
1 hour later…
2:56 PM
@XanderHenderson You are aware of the framing effect, right? I'll just quote Gerry's comment, which I agree with:
@213, it's one thing when a student wants help with a Calculus exercise. It's a very different thing when someone thinks he/she presents a "proof" of a notorious open question, using nothing beyond high school math. If you're going to do math research, you have to learn to do it the way mathematicians do it, which is to be your own fiercest critic, searching out the weakest points in your argument, finding your mistakes before anyone else does. Shooting down feeble attempts at major questions is definitely not the point of this website. — Gerry Myerson 4 hours ago
I am of the opinion that the framing effect is the biggest factor in explaining behaviour on Meta, but I also know that there's nothing much we can do about it.
 
 
1 hour later…
4:16 PM
1
Q: Is every large enough odd integer the sum of a prime or prime power and a power of $2$?

Sylvain JulienI've been thinking for some months about a slightly weaker form of Goldbach's conjecture: namely that every large enough integer is the sum of two prime powers or primes, that is $\exists C>0,\forall n>C,n=p^a+q^b$ with $p$ and $q$ primes and $a$ and $b$ positive integers. Of course if $n$ is odd...

 
 
3 hours later…
7:16 PM
PSQ? Unclear? There are a couple of answers which (essentially) start "Not an answer, but..." Evaluating a special case of $\frac{\sum_{j=1}^n k!}{ (n+1)!}$‭ - DatBoi‭ 2022-05-27 05:18:47Z
 
7:54 PM
this answer is not an answer at all: math.stackexchange.com/a/4460096/348926
 
 
3 hours later…
11:02 PM
[ SmokeDetector | MS ] Obfuscated word in body (97): Airfare Ticket Reduction Optimization Problem‭ by thekerbal‭ on math.SE
 

« first day (3788 days earlier)      last day (718 days later) »